Maths, would it be congruent or alternate??

Maths, Would It Be Congruent Or Alternate??

Answers

Answer 1

Therefore , the solution of congruency problem is its two congruent halves are LJ and LI. The sections' sizes and forms must be consistent to be considered congruent.

What is congruence?

Using the Side-Angle-Side Congruity Theorem (SAS), two triangles are believed to be compatible with one another if their corresponding two sides and included angle are the same. A defined angle is found to be present on 2 sections that are being taken into account. Therefore, it is insufficient to use the SAS Congruity Theorem to establish the congruence of two triangles based just on the presence of two pairs of reciprocals and one set of points of intersection.

Here,

Given: The illustration shows a congruent link between

the two portions LI and LJ.

The segments will be identical in terms of size and shape if they are congruent.

To know more about congruency visit :-

brainly.com/question/2938476

#SPJ1


Related Questions

A dairyman wishes to mix milk containing 5% butterfat and cream containing 75% butterfat to produce a total mixture of 56 liters. This final mixture should contain 53% butterfat. How much of the milk mixture and how much of the cream mixture should he use

Answers

18% of the milk mixture and how much of the cream mixture should he use.

How much of the milk mixture and how much of the cream mixture should he use?

A dairyman wants to combine milk with 5 percent butterfat and cream with 75 percent butterfat to create a 56-liter combination. Butterfat should make up 53% of the final combination.

Let x represent the volume of MILK required for the combination, in liters.

Consequently, x/56 is the PROPORTION of milk in the mixture. [because the final mixture has 56 liters in total]

We require 56 - x liters of CREAM in the mixture because we have 56 liters overall in the mixture.

The PROPORTION of cream in the combination is therefore equal to (56 - x)/56.

Our goal is for the final mixture to have 75% butterfat.

Fill in the equation with each of these values to obtain:

50 = (x/60)(5) + ((60 - x)/60) (75)

Add 56 to both sides to get: 3000 = (5)(x) + (56 - x)(75)

The formula is:

Multiply both sides by 56 to get: 3000 = (5)(x) + (56 - x)(75)

Expand: 3000 = 5x + 4200 - 75x

Simplify: 3000 = 4200 - 70x

Subtract 4500 from both sides: -1300 = -70x

Solve: x = (-1300)/(-70) = (1300)/(70) = 130/7

If you don't want to divide 130 by 7, you can evaluate this quickly by first realizing that 30/7 = 18.

Consequently, 130/7 must be a little larger than 18.

Learn more about milk mixture here:

https://brainly.com/question/11026475

#SPJ4

x-15/3 = 18x. How to solve this question? I will mark the first answerer as brainliest.

Answers

Solution: -15/53

Explanation:

X-15/3 = 18x

Multiply both sides by 3
(x-15/3) * 3 = (18x) * 3
x - 15 = 54x

Subtract x from the left hand side
x-x-15 = 54x
-15 = 54x - x
-15 = 53x

Divide both sides by 53
-15/53 = 53x/53

-15/53 = x

Additionally, mathematical logic is a subfield of mathematics exploring the applications of formal logic to mathematics. In your initial post for this discussion, address both of the following:What are differences or similarities between everyday logic and mathematical logic?How can the study of mathematical logic help you in your everyday life?

Answers

Mathematical logic is important as it's a way to learn new experience through continuous self assessment.

How to illustrate the information?

Logic is important as it enables us to form sound judgements and beliefs.

The study of logic can help as it can help us to understand disagreement and ambiguity.

It also helps us in making a reasonable emotional life.

Learn more about logic on:

brainly.com/question/24912495

#SPJ1

1. Reflect Rectangle ABCD across the y-axis, then
translate it using the following rule:
(x,y) → (x-4, y - 3).
A B
D
с
-4 -2
4
2
ТУ
O
-2-
-4-
2
4
X
√x
A' (
B'(
C'(
D'(
A"(
B"(
C"(
D"(
)
)
)
)
PLS HELP ME WITH THIS

Answers

Answer:

See attached image

Step-by-step explanation:

Explain how you can find the constant of proportionality from a graph representing a proportional relationship when it shows a point with an x-value of 1 and if it doesn’t show an x-value of 1.

Answers

For a proportional relationship, the constant is found dividing all values of y by each respective value of x.

What is a proportional relationship?

A proportional relationship is a function in which the output variable is given by the input variable multiplied by a constant of proportionality, that is:

y = kx

In which k is the constant of proportionality.

The constant can be represented as follows:

[tex]k = \frac{y}{x}[/tex]

Hence the constant is found dividing all values of y by each respective value of x.

More can be learned about proportional relationships at https://brainly.com/question/10424180

#SPJ1

A farmer uses a lot of fertilizer to grow his crops. The farmer's manager thinks fertilizer products from distributor A contain more of the nitrogen that his plants need than distributor B's fertilizer does. He takes two independent samples of four batches of fertilizer from each distributor and measures the amount of nitrogen in each batch. Fertilizer from distributor A contained 33 pounds per batch and fertilizer from distributor B contained 25 pounds per batch. Suppose the population standard deviation for distributor A and distributor B is three pounds per batch and four pounds per batch, respectively. Assume the distribution of nitrogen in fertilizer is normally distributed. Let µ1 and µ2 represent the average amount of nitrogen per batch for fertilizer A and B, respectively. Which of the following is the correct value of the test statistic?

Answers

Based on the calculations, the correct value of the test statistic is equal to 3.2.

How to calculate value of the test statistic?

For samples A and B, the hypothesis is given by:

H₀: μ₁ ≤ μ₂

H₁: μ₁ > μ₂

Since both samples have a normal distribution, we would use a pooled z-test to determine the value of the test statistic:

[tex]z = \frac{\bar{x_1} - \bar{x_2} -(\mu_1 - \mu_2) }{\sqrt{\frac{\sigma_1^2}{n_1} + \frac{\sigma_1^2}{n_1}} }[/tex]

Substituting the given parameters into the formula, we have;

[tex]z = \frac{33 - 25 -(0) }{\sqrt{\frac{3^2}{4} + \frac{4^2}{4}} }\\\\z = \frac{8 }{\sqrt{\frac{9}{4} + \frac{16}{4}} }\\\\z = \frac{8 }{\sqrt{\frac{25}{4} } }\\\\z = \frac{8 }{\frac{5}{2} }}\\\\z = 8 \times \frac{2}{5}[/tex]

z = 3.2.

Read more on standard deviation here: brainly.com/question/4302527

#SPJ1

Teboho has to sell a certain number of cars each month. For every car he sells over and above this number he earns 17% commission on the price of the car before VAT is added. He sold three cars on which he earned commission. The VAT included prices were R271 800,00; R316 800,00 and R99 300,00. Calculate:

1.1 The prices of the cars before VAT was added

1.2 The commission he earned on the cars.​

Answers

The commission earned are R41123.17, R47931.67 and R15024.09

The prices of the cars before VAT was added

The VAT included prices are given as:

R271 800,00; R316 800,00 and R99 300,00.

The VAT rate is 12.36%.

So, the price before VAT is calculated as:

VAT included = Price without VAT * (1 + 12.36%)

This gives

VAT included = Price without VAT * 1.1236

So, we have:

Price without VAT = VAT included/1.1236

For the three cars, we have:

Price without VAT = R271 800.00/1.1236 = R241901

Price without VAT = R316800.00 /1.1236 = R281951

Price without VAT = R99300.00/1.1236 = R88377

Hence, the prices of the cars before VAT was added are R241901, R281951 and R88377

The commission he earned on the cars.​

This is calculated as:

Commission = Price * 17%

So, we have:

Commission = R241901 * 17% = R41123.17

Commission = R281951 * 17% = R47931.67

Commission = R88377 * 17% = R15024.09

Hence, the commission earned are R41123.17, R47931.67 and R15024.09

Read more commission at:

https://brainly.com/question/20987196

#SPJ1

How many palindromes greater than 10000 and less than 100000 are multiples of 18?

Answers

Let [tex]n=abcba[/tex] be such a number. If 18 divides [tex]n[/tex], then both 2 and 9 divide [tex]n[/tex].

To be divisible by 2, we must have [tex]a\in\{2,4,6,8\}[/tex]. Meanwhile we can have [tex](b,c)\in\{0,1,2,\ldots,9\}^2[/tex].

To be divisible by 9, we the sum of the digits of [tex]n[/tex] must itself be divisible by 9, or

[tex]2a+2b+c=9k[/tex]

for some integer [tex]k[/tex].

The largest value of [tex]2a+2b+c[/tex] is 2•8 + 2•9 + 9 = 43, so we must have [tex]k\in\{1,2,3,4\}[/tex].

I'm not sure what the best way to get the final count may be, but there are 44 such numbers. It's rather tedious to do by hand.

• If [tex]k=1[/tex], then [tex]2a+2b+c=9[/tex], and we can do this in 4 ways.

For example,

2•2 + 2•0 + 5 = 9 [tex](n = 20502)[/tex]

• If [tex]k=2[/tex], then [tex]2a+2b+c=18[/tex] and can be done in 16 ways.

2•2 + 2•3 + 8 = 18 [tex](n = 23832)[/tex]

• If [tex]k=3[/tex], then [tex]2a+2b+c=27[/tex] and can be done in 18 ways.

2•2 + 2•7 + 9 = 27 [tex](n = 27972)[/tex]

• If [tex]k=4[/tex], then [tex]2a+2b+c=36[/tex] and can be done in 6 ways.

2•6 + 2•8 + 8 = 36 [tex](n = 68886)[/tex]

Mark each statement as true or false. Suppose A is an n n matrix. a. If an n × n matrix A has fewer than n distinct eigenvalues, then A is not diagonalizable. False b. If A is diagonalizable, then A is also diagonalizable False c. If there is a basis of R n consisting of eigenvectors of A, then A is diagonalizable. True d. A is diagonalizable if and only if A has n eigenvalues, counting multiplicity. False e. If A is diagonalizable, then A is invertible. False

Answers

The correct option for the matrix will be:

FalseTrueTrueFalseFalse

How to explain the matrix?

a) If an n x n matrix A has fewer than n distinct eigenvalues, then A is not diagonalizable.

False

It could have repeated eigenvalues as long as the basis of each eigenspace is equal to the multiplicity of that eigenvalue.

b) If A is diagonalizable the A2 is diagonalizable

True

If A is diagonalizable then there exists an invertible matrix

c) If Rn has a basis of eigenvectors of A, then A is diagonalizable.

True

d) A is diagonalizable if and only if A has n eigenvalues, counting multiplicity.

False

e) If A is diagonalizable, then A is invertible.

False

It’s invertible if it doesn’t have a zero as eigenvalue but this doesn’t affect diagonalizable.

Learn more about matrix on:

https://brainly.com/question/1821869

#SPJ1

Triangle ΔABC is reflected across line n to create ΔA'B'C'
What is the measure of ∠C?

Answers

Answer:

54 degrees

Step-by-step explanation:

The actual angles of the triangle are not changing since this is just a reflection. The angles inside a triangle all add up to 180, so we can do 180-59-67 to get 54.

100 families booked a holiday in July or in August, at travel agents.
Some of the families booked to go to France.
Some booked to go to Spain.
The rest of the families booked a holiday to Portugal.
59 families booked to go on holiday in August. 19 of the 35 families going to France booked to go in July.
30 families booked to go to Portugal.
20 families booked to go to Spain in August. How many families booked to go to Portugal in July?​

Answers

Answer:

  7

Step-by-step explanation:

A 2-way table can be useful for recording the given information and for finding the missing numbers.

Setup

The attached table shows the numbers of families booking in July and August, and also counted by destination. Totals are at the right and bottom, and the grand total is the number 100 at the lower right.

Underlined numbers are those in the problem statement. The remaining numbers are computed so as to make the totals be correct.

Solution

Overall, 30+35 = 65 went to Portugal or France, so 100-65 = 35 went to Spain. Of those, 20 booked in August, so 15 booked to Spain in July.

59 booked in August, so a total of 41 booked in July.

Now we know 19 booked for France and 15 booked for Spain in July, so the remaining 7 booked for Portugal in July.

Solve the system of equations 4x+5y=-1 and -5x-8y=10 by combining the equations.

Answers

The solution of the equation are as follows:

x = 6 and y = -5

How to solve the system of equation?

4x + 5y = -1

-5x - 8y = 10

Therefore,

20x + 25y = -5

-20x - 32y = 40

-7y = 35

y = -5

Hence,

4x + 5(-5) = -1

4x - 25 = -1

4x = -1 + 25

4x = 24

x = 24 / 4

x = 6

learn more on equation here:https://brainly.com/question/14383993

#SPJ1

 What is the sum of the terms of the series 1+3+5+...+15

Answers

1+3+5+7+9+11+13+15

=4+12+20+28

=16+48

=64

PLEASE HELP!!! I'LL MARK AS BRAINLIEST TO THE FIRST PERSON THAT CAN ANSWER!!!

When two exponents with the same base are multiplied together, this reflects the _______ of powers property.

Answers

The product of powers property.

Select the correct answer from the drop-down menu.
The expression
is not equivalent to (1 - sin²(x)) tan(-x).
J
e
C

Answers

The expression that is not equivalent to (1 - sin²(x)) tan(-x) is D. (cos²x - 1)(cot -x).

How to illustrate the expression?

It should be noted that (cos²x - 1)(cot -x). is not equivalent to the given expression.

This is illustrated as:

(cos²x - 1)(cot -x)

= (-sin²x) × (-cot x)

= sin²x × cosx/sin x = sinxcosx

In conclusion, the correct option is D.

Learn more about expressions on:

brainly.com/question/12651505

#SPJ1

The expression _______ is not equivalent to (1 − sin2(x)) tan(-x).

a. (1 - cos^2(x)) cot(-x)

b. (cos^2(x) - 1) cot(x)

c. (sin^(x) - 1) tan(x)

d. (cos^2(x) - 1) cot(-x)

Which represents the solution set to the inequality 5.1(3 + 2.2x) > –14.25 – 6(1.7x + 4)?
x < –2.5
x > 2.5
(–2.5, ∞)
(–∞, 2.5)

Answers

The solution to the inequality is (–2.5, ∞)

How to solve the inequality?

The inequality is given as:

5.1(3 + 2.2x) > –14.25 – 6(1.7x + 4)

Open the brackets

15.3 + 11.22x > –14.25 – 10.2x - 24

Collect the like terms

11.22x + 10.2x > -15.3 - 14.25- 24

Evaluate the like terms

21.42x > -53.55

Divide both sides by 21.42

x > -2.5

This can also be represented as (–2.5, ∞)

Hence, the solution to the inequality is (–2.5, ∞)

Read more about inequality at

https://brainly.com/question/24372553

#SPJ1

Answer:

c

Step-by-step explanation:

on edge

Right triangles \boxed{1}
1

start box, 1, end box, \boxed{2}
2

start box, 2, end box, and \boxed{3}
3

start box, 3, end box are given with all their angle measures and approximate side lengths.








Use one of the triangles to approximate the ratio \dfrac{WY}{WX}
WX
WY

start fraction, W, Y, divided by, W, X, end fraction.

Choose 1 answer:

Answers

Considering right-angled triangle XYW, an approximate ratio WY/WX is equal to: A. 0.34.

How to approximate the ratio WY/WX?

Considering right-angled triangle XYW, we would apply the law of cosine to approximate the ratio WY/WX. Mathematically, the law of cosine is given by:

cos(θ) = Adj/Hyp

Where:

Adj is the adjacent side of a right-angled triangle.Hyp is the hypotenuse of a right-angled triangle.θ is the angle.

Substituting the given parameters into the formula, we have;

cos(θ) = WY/WX

cos(70) = 3.4/10

0.34 = 0.34.

Read more on triangle ratio here: https://brainly.com/question/26060091

#SPJ1

Complete Question:

Right triangles 1-2 and 3 are given with all their angle measures and approximate side lengths.

Use one of the triangles to approximate the ratio WY/WX

A. 0.34

B. 0.94

C. 1.06

D. 2.76​

Evaluate the interval (Calculus 2)

Answers

Answer:

[tex]2 \tan (6x)+2 \sec (6x)+\text{C}[/tex]

Step-by-step explanation:

Fundamental Theorem of Calculus

[tex]\displaystyle \int \text{f}(x)\:\text{d}x=\text{F}(x)+\text{C} \iff \text{f}(x)=\dfrac{\text{d}}{\text{d}x}(\text{F}(x))[/tex]

If differentiating takes you from one function to another, then integrating the second function will take you back to the first with a constant of integration.

Given indefinite integral:

[tex]\displaystyle \int \dfrac{12}{1-\sin (6x)}\:\:\text{d}x[/tex]

[tex]\boxed{\begin{minipage}{5 cm}\underline{Terms multiplied by constants}\\\\$\displaystyle \int a\:\text{f}(x)\:\text{d}x=a \int \text{f}(x) \:\text{d}x$\end{minipage}}[/tex]

If the terms are multiplied by constants, take them outside the integral:

[tex]\implies 12\displaystyle \int \dfrac{1}{1-\sin (6x)}\:\:\text{d}x[/tex]

Multiply by the conjugate of 1 - sin(6x) :

[tex]\implies 12\displaystyle \int \dfrac{1}{1-\sin (6x)} \cdot \dfrac{1+\sin(6x)}{1+\sin(6x)}\:\:\text{d}x[/tex]

[tex]\implies 12\displaystyle \int \dfrac{1+\sin(6x)}{1-\sin^2(6x)} \:\:\text{d}x[/tex]

[tex]\textsf{Use the identity} \quad \sin^2 x+ \cos^2 x=1:[/tex]

[tex]\implies \sin^2 (6x) + \cos^2 (6x)=1[/tex]

[tex]\implies \cos^2 (6x)=1- \sin^2 (6x)[/tex]

[tex]\implies 12\displaystyle \int \dfrac{1+\sin(6x)}{\cos^2(6x)} \:\:\text{d}x[/tex]

Expand:

[tex]\implies 12\displaystyle \int \dfrac{1}{\cos^2(6x)}+\dfrac{\sin(6x)}{\cos^2(6x)} \:\:\text{d}x[/tex]

[tex]\textsf{Use the identities }\:\: \sec \theta=\dfrac{1}{\cos \theta} \textsf{ and } \tan\theta=\dfrac{\sin \theta}{\cos \theta}:[/tex]

[tex]\implies 12\displaystyle \int \sec^2(6x)+\dfrac{\tan(6x)}{\cos(6x)} \:\:\text{d}x[/tex]

[tex]\implies 12\displaystyle \int \sec^2(6x)+\tan(6x)\sec(6x) \:\:\text{d}x[/tex]

[tex]\boxed{\begin{minipage}{5 cm}\underline{Integrating $\sec^2 kx$}\\\\$\displaystyle \int \sec^2 kx\:\text{d}x=\dfrac{1}{k} \tan kx\:\:(+\text{C})$\end{minipage}}[/tex]

[tex]\boxed{\begin{minipage}{6 cm}\underline{Integrating $ \sec kx \tan kx$}\\\\$\displaystyle \int \sec kx \tan kx\:\text{d}x= \dfrac{1}{k}\sec kx\:\:(+\text{C})$\end{minipage}}[/tex]

[tex]\implies 12 \left[\dfrac{1}{6} \tan (6x)+\dfrac{1}{6} \sec (6x) \right]+\text{C}[/tex]

Simplify:

[tex]\implies \dfrac{12}{6} \tan (6x)+\dfrac{12}{6} \sec (6x)+\text{C}[/tex]

[tex]\implies 2 \tan (6x)+2 \sec (6x)+\text{C}[/tex]

Learn more about indefinite integration here:

https://brainly.com/question/27805589

https://brainly.com/question/28155016

Substitute [tex]y=6x[/tex] and [tex]dy=6\,dx[/tex] to transform the integral to

[tex]\displaystyle \int \frac{12}{1-\sin(6x)} \, dx = 2 \int \frac{dy}{1 - \sin(y)}[/tex]

Now substitute [tex]t=\tan\left(\frac y2\right)[/tex] and [tex]dt=\frac12 \sec^2\left(\frac y2\right) \, dy[/tex] to transform this to

[tex]\displaystyle 2 \int \frac{dy}{1 - \sin(y)} = 2 \int \frac1{1-\frac{2t}{1+t^2}}\cdot\frac{2\,dt}{1+t^2} = 4 \int \frac{dt}{(t-1)^2}[/tex]

Finally, substitute [tex]s = t-1[/tex] and [tex]ds=dt[/tex] to get

[tex]\displaystyle 4 \int \frac{dt}{(t-1)^2} = 4 \int \frac{ds}{s^2} = -\dfrac4s + C[/tex]

Now recover the antiderivative in terms of [tex]x[/tex].

[tex]\displaystyle \int \frac{12}{1-\sin(6x)} \, dx = -\frac4s + C \\\\ ~~~~~~~~ = -\frac4{t-1} + C \\\\ ~~~~~~~~ = -\frac4{\tan\left(\frac y2\right) - 1} + C \\\\ ~~~~~~~~ = \boxed{-\frac4{\tan(3x) - 1} + C}[/tex]

A rectangular closet has a perimeter of 18 feet and an area of 20 square feet. What are the dimensions of the closet.

Answers

Answer:

The width can either be 4 or 5 feet.

Step-by-step explanation:

Assuming the length is x and the width is y.2x+2y=18xy=202y=18−2xy=9−xx(9−x)=209x−x2=200=x2−9x+200=(x−5)(x−4)x=5and4 The width can either be 4 or 5 feet

A 12-ounce cup of juice contains 70 percent fruit and 30 percent water. If combined with 16 ounces of juice that contain 10 percent fruit and 90 percent water, what portion of the mixture is fruit?
Write the answer to the nearest hundredth

Answers

The portion of the mixture that is fruit to the nearest hundredth is 10.00 ounces

Percentage

12 ounce juice:

Water = 30%Fruits = 70%

= 70/100 × 12

= 0.7 × 12

= 8.4 ounces

Water = 30%

16 ounces;

Water = 90%Fruits = 10%

= 10/100 × 16

= 0.1 × 16

= 1.6 ounces

Portion of mixture that is fruits = 8.4 ounces + 1.6 ounces

= 10.00 ounces

Learn more about percentage:

https://brainly.com/question/843074

#SPJ1

A basketball player has a 0.689 probability of making a free throw. If the player shoots 18 free throws, what is the probability that she makes no more than 11 of them

Answers

It is determined while using the binomial distribution that there is still a 1.145=114.5% chance that she produces no more than 11 of them.

Calculating the probability

There are just two possible results for each throw. Either she succeeds or she fails. The binomial probability distribution is employed to answer this issue since the probability of completing a shot is regardless of all other throws.

Binomial probability distribution-

[tex]P(X=x) = C_{n,x} .p^{x}(1-p)^{n-x}[/tex]

[tex]C_{n,x} = n!/x! (n-x)![/tex]

where,

the no. of success= x

the no. of trials = n

the probability of a success on one trial = p

The probability of throwing not more than 11 will be:

P(X<11) = P(X=0) + P(X=1) +P(X=2) + P(X=3) +P(X=4) +P(X=5)+P(X=6)+P(X=7)+P(X=8)+P(X=9)+P(X=10)+P(X=11)

Where,

[tex]P(X=x) = C_{n,x} .p^{x}(1-p)^{n-x}[/tex]

[tex]P(X=0) = C_{18,0} .(0.689)^{0}(0.311)^{18}[/tex]≈0

[tex]P(X=1) = C_{18,1} .(0.689)^{1}(0.311)^{17}[/tex]≈0

[tex]P(X=2) = C_{18,2} .(0.689)^{2}(0.311)^{16}[/tex]≈0

[tex]P(X=3) = C_{18,3} .(0.689)^{3}(0.311)^{15}[/tex]≈0

[tex]P(X=4) = C_{18,4} .(0.689)^{4}(0.311)^{14}[/tex]≈0

[tex]P(X=5) = C_{18,5} .(0.689)^{5}(0.311)^{13}[/tex]= 0.0003

[tex]P(X=6) = C_{18,4} .(0.689)^{6}(0.311)^{12}[/tex]=0.0016

[tex]P(X=7) = C_{18,7} .(0.689)^{7}(0.311)^{11}[/tex]=0.0062

[tex]P(X=8) = C_{18,8} .(0.689)^{8}(0.311)^{10}[/tex]=0.0188

[tex]P(X=9) = C_{18,9} .(0.689)^{9}(0.311)^{9}[/tex]=0.0463

[tex]P(X=10) = C_{18,10} .(0.689)^{10}(0.311)^{8}[/tex]=0.9232

[tex]P(X=11) = C_{18,11} .(0.689)^{11}(0.311)^{7}[/tex]=0.1488

So,

P(X<11) = P(X=0) + P(X=1) +P(X=2) + P(X=3) +P(X=4) +P(X=5)+P(X=6)+P(X=7)+P(X=8)+P(X=9)+P(X=10)+P(X=11)

=0+0+0+0+0+0.0003+0.0016+0.0062+0.0188+0.0463+0.9232+0.1488 =1.145

Therefore, she makes 1.145=114.5% probability, no more than 11 of them.

Learn more about probability here:

https://brainly.com/question/11234923

#SPJ4

Ralph Warren purchased 27 shares of stock at 16 3/8 per share. He paid a $27.50 brokerage fee. He later sold all 27 shares at 17 5/8 and paid a $28.75 brokerage fee. (36) What was his total cost for the stock including his brokerage fee? (37) What did he receive from the sale of the stock after he paid the brokerage fee? (38) Did he have a capital gain or loss? (39) How much was the gain or loss? (40) What was the net change from 16 3/8 to 17 5/8?

Answers

Answer:

Ralph Warren purchased 27 shares of stock at 16 3/8 per share. He paid a $27.50 brokerage fee. He later sold all 27 shares at 17 5/8 and paid a $28.75 brokerage fee. (36) What was his total cost for the stock including his brokerage fee? (37) What did he receive from the sale of the stock after he paid the brokerage fee? (38) Did he have a capital gain or loss? (39) How much was the gain or loss? (40) What was the net change from 16 3/8 to 17 5/8?

Step-by-step explanation:

B(x) = 0. 06x^2 - 0. 2x^3, find the dosage at which the resulting blood presure is maximized

Answers

The dosage at which the resulting blood pressure is maximized is x at 0.2 and the maximum dosage is 0.0008.

In the question,

The function is [tex]B(x) = 0. 06x^2 - 0. 2x^3[/tex]

To find the maximum or minimum, take the derivative and set it equal to zero.

⇒ [tex]B'(x) = 2(0. 06)x - 3(0. 2)x^{2}[/tex]

Setting it equal to zero, we get

⇒ [tex]0.12x - 0.6x^{2}=0[/tex]

⇒ 0.6x (0.2-x) = 0

⇒ x = 0 or x = 0.2

Now substitute x = 0.2 in B(x), we get

⇒ [tex]B(0.2) = 0. 06(0.2)^{2} - 0. 2(0.2)^{3}[/tex]

⇒ B(0.2) = 0.0024 - 0.0016

⇒ B(0.2) = 0.0008

To know B(0.2) is maximum, let us find the values for x = 1 and x = 0.01.

For x = 1,

⇒ [tex]B(1) = 0. 06(1)^{2} - 0. 2(1)^{3}[/tex]

⇒ B(1) = 0.06-0.2

⇒ B(1) = -1.04

For x = 0.01,

⇒ [tex]B(0.01) = 0. 06(0.01)^{2} - 0. 2(0.01)^{3}[/tex]

⇒ B(0.01) = 0.000006 - 0.0000002

⇒ B(0.01) = 0.0000058

Thus, x = 0.2 is the maximum.

Hence we can conclude that the dosage at which the resulting blood pressure is maximized is x at 0.2 and the maximum dosage is 0.0008.

Learn more about function here

https://brainly.com/question/12913444

#SPJ4

Which numbers are integers? Check all that apply.
4
Negative 1 and one-third
-10
2.5
-4
0.ModifyingAbove 13 with Bar





YOOOO QUICK I REALLY NEED THIS PLEASEEE

Answers

The numbers : 4,-10 and -4 are integers.

A number without a decimal or fractional element is known as an integer, which can be both positive and negative, including zero.

The Latin word "integer" signifies "whole" or "intact." Thus, fractions and decimals are not included in integers.

All whole numbers and negative numbers are considered integers. This means that if we combine negative numbers with whole numbers, a collection of integers results.

"Negative 1 and one-third" includes a fractional part, so it is not an integer.

"2.5" and "0.ModifyingAbove 13 with Bar" contain a decimal, so it is also not an integer.

Learn more about integers here:

https://brainly.com/question/12683055

#SPJ1

There are Z fish in a aquarium. 1/4 of the fish are angelfish. How may are angelfish?

Answers

Step-by-step explanation:

since there are z fishes

the number of angelfish = 1 × z

4

= 1 z

4

The answer is 1/4*z=1/4z

The acting club two-act plays begins at 3:20 P.M. The first act is twice as long as the second act, and there is a 15-minute break between the two acts. The play ends at 4:50 P.M. How long is Act 1?

Answers

Answer:

50 minutes

Step-by-step explanation:

3:20 pm - 4:50 = 90 minutes

90 = 2a + 15 + a

90-15= 75

75= 2a + a

75 = 3a

75/3 = 25

25 x 2 = 50 minutes

Find the discriminant of the quadratic equation x2 6x 14 = 0 and use it to determine the number and types of solutions. b2 − 4ac −20; two nonreal solutions −20; one real solution 92; two real solutions 92; one real solution

Answers

The discriminant of the quadratic equation is -20 and there are two non real solutions.

The discriminant of a quadratic equation uses the equation:

[tex]b^{2} -4ac[/tex]

Where the value of this calculation can tell you what solutions there are, plug known values in:

[tex]x^{2} +6x+14[/tex]

a=1

b=6

c=14

[tex]b^{2} -4ac[/tex]

which is equal to -20 on putting the values of a,b and c in the equation

As -20 < 0, this means that there is not a real solution, resulting in the first option being correct.

For more information about discriminant, visit https://brainly.com/question/2507588

#SPJ1

PLEASE HELP!!!! I don’t understand this topic please can someone help explain how to work out the attached question on similar area. Will mark brainliest!

Answers

Step-by-step explanation:

Look, you're taking these kinds of questions too serious and that's why you believe they're hard, now pay close attention:

the areas are defined in cm² but the radius is defined in cm, so you need find the positive root of cm².

[tex] \sqrt{ \frac{5500 {cm}^{2} }{220 {cm}^{2} } } = \frac{r}{5cm} = = > \\ 5 = \frac{r}{5} = = = > r = 25[/tex]

and the question wants the base area:

[tex]s = \pi {r}^{2} = = = > s = 3.14 \times {(25cm)}^{2} = = > 1963.5 {cm}^{2} [/tex]

A basin can hold 8542ml of water.the basin can hold 0.458l more water than a fish tank. how much water can the fish tank hold?express your answer in litres.

Answers

Answer:

8.048 liters

Step-by-step explanation:

Convert 8542 ml to liters:  (8542 ml)(1 liter/1000 ml) = 8.542 liters

Now subtract 0.458 liters to find the amount that a fish tank can hold:

 8.542 liters

-0.458  liters

8.048 liters is the fish tank capacity

In a bag of 10 marbles, there are 4 blue, 3 red, 2 green,
and 1 yellow. What is the probability that you draw one
marble that is red, replace it, and draw another marble that
is yellow?
a. 2
-
5
b. 3
100
C. 1
30
O d. 3
10

Answers

[tex]P(R \: then \: Y) = \frac{3}{10} \times \frac{1}{10} = \frac{3}{100} [/tex]

[tex]note \: that = \\ order \: matters \: (no \: factorial) \\ replacement \: (equal \: sample \: space)[/tex]

Option B

Answer:

c. 3/100

Step-by-step explanation:

there are 10 marbles

First draw (there are 3 red marbles)

[tex]P=\frac{3}{10}[/tex]

Second draw (there is one yellow marble)

[tex]P=\frac{1}{10}[/tex]

probability of the event:

[tex]P=(\frac{3}{10} )(\frac{1}{10} )=\frac{3}{100}[/tex]

Hope this helps

Other Questions
Give a algebra equation to represent this pattern Sensory receptors Categorize the following based on the type of sensory receptor The length of a rectangle is 4yd longer than its width. If the perimeter of the rectangle is 40yd , find its area. HELP THIS IS DUE TOMORROW!!!! Since the fourth quarter of 2014, the flash storage capacity of smartphones has grown by approximately 12 gigabytes a year. The average flash storage in 2014 was 15.9 gigabytes. a. Write an equation to represent this situation , defining both variables . b . Assuming the trend continues , write a one - varia equation that can be used to find the average f storage capacity of smartphones in 2025 . C. Solve your equation from Part B.D. Interpret the solution What are the 4 coordinates of a graph? What is the fewest number of regions that three lines can separate a plane into? Describe the configuration of lines that gives you this many regions. What is the most common reason why countries create trade agreements brainly? How did the criteria of realist writers such as Stephen crane differ from the standards of romantic writers of the past?Why was realism well suited to describe the horrors of war?Why might nineteenth-century readers have been especially receptive to regionalist and local-color writing?In your opinion, which qualities of regionalism might most appeal to todays readers?The Rise of Realism what does -2/3 slope look like on a graph Which biotechnology R&D professionals are certified by the National Accrediting Agency for Clinical Laboratory Services (NAACLS)?O A. physiologistsOB.O C.OD. biochemistsbiological techniciansprocess technicians The nazi plan to murder the entire jewish population of europe and the soviet union was known as the write the formula for isothermal compressibility (see example 5) in terms of p Which type of initiative makes it more profitable to follow a certain course of action? Which equation could be used to find the missing side of the triangle shown?10 mam5 mO a +5210O 5 + 10 aO2 (5) +2 (10) = 2aO a + 10 5= What specific year does the video differentiate the two new deals from one another? If I see the star Sirius in the constellation of Canis Major rise at 8 pm tonight, what time will it rise one week from now The term 'approved suppliers' means a foodservice operation can buy only from national vendors. a) True b) False how many amendments does the texas constitution have A train moves from rest to a speed of 25 m/s in 30.0 seconds. What is the magnitude of it's acceleration? the coach from team a submits the line-up for set one with #9 designated as the libero. who can the coach designate as a libero for the second set?